RegistrierenRegistrieren   LoginLogin   FAQFAQ    SuchenSuchen   
Widerstandswürfel mit unterschiedlichen Widerständen
 
Neue Frage »
Antworten »
    Foren-Übersicht -> Elektrik
Autor Nachricht
Nice_price



Anmeldungsdatum: 05.09.2010
Beiträge: 4

Beitrag Nice_price Verfasst am: 05. Sep 2010 17:59    Titel: Widerstandswürfel mit unterschiedlichen Widerständen Antworten mit Zitat

Meine Frage:
Moin moin liebe Community,
es ist mal wieder so weit, eine Klausurersatzleistung steht an. Diesmal geht es um einen Widerstandswürfel oder besser gesagt um 2.
Zeichnungen findet man z.B. hier:
http://www.lehrer.uni-karlsruhe.de/~za275/ph12/aufgaben/01_aufloes_widerstandswuerfel.pdf

Der erste Würfel hat tatsächlich 12 Widerstände zu je 100 Ohm, der bereitet mir in einer Kleinigkeit schwierigkeiten, das ist jedoch das kleinere Problem: Mit den Punkten A und G ist meine Stromquelle verbunden (2V;0,025A). Nachdem ich verschiedene Stromstärken und den Gesamtwiderstand berechnet habe soll ich zu guter Letzt noch die Spannung über Punkt A und Punkt H messen und berechnen. Die Messung ergab 1,2V.

Das zweite Problem ist etwas komplexer;)
Ich habe auch noch einen zweiten Würfel zusammenlöten sollen. Habe ich auch gemacht und durchgemessen, der ist allerdings folgendermaßen aufgebaut: Die KAnten die von dem Punkt A abgehen haben Widerstände von je 200 Ohm, die drei, die vom Punkt G abgehen alle 300 Ohm und der Rest liegt bei 100 Ohm. Hier muss ich sowohl verschiedene Stromstärken messen und berechnen. (Zwischen A und E bzw. A und H[wie oben]) (gemessen: 3,6mA bzw. 1,8mA oder 3,7mA --> da hat der Kollege die beiden Werte hingeschrieben als ich gemessen habe, keine Ahnung wieso)
Außerdem soll ich aber auch hier den Gesamtwiderstand berechnen, der mit meinen gemessenen ca.180 Ohm aber nicht so wirklich richtig zusammenpasst.Das Netzgerät zeigte bei diesem Würfel bei den geforderten 2V auch nur noch eine Stromstärke von 0,01A an.

Ich komm nicht weiter, vielleicht rettet ihr ja den Sonntag=) Oder den Anfang der Woche;)
DANKESCHÖN!!

Meine Ideen:
Zu dem ersten Problem: In dem Link ist der einfachste Weg beschrieben den Gesamtwiderstand zu berechnen. Dafür ist die Spannung auf einem der Wege von A nach G berechnet (R*I*(1/3+1/3+1/6)).Ich hab versucht mir das so umzubauen, dass es passt. Funktioniert aber nicht, kommen 1,67 Volt bei raus, nicht 1,2.... Auch googeln brachte mich nur auf Äquipotentialpunkte, aber ich habe davon noch nie gehört und googeln brachte auch keine vernünftige Erklärung... Hilfe?!? Also das googeln brachte mich dann (nebenbei bemerkt) darauf, dass der Widerstand auf diesen beiden Ästen bei 3/4 R (also 75 Ohm) liegen muss. Wie man darauf kommt ist mir aber schleierhaft... Um damit dann rechnen zu können bräuchte ich auch die Stromstärke, auf dem ersten Ast (an A) liegen 1/3I, auf dem 2. dann ja nur noch 1/6I, mit welcher muss ich denn da dann rechnen?

Zum zweiten:
Stundenlang gegoogelt und nichts gefunden, ich krieg den Widerstandswürfel aber mit den in den Link genannten Formeln nicht in den Griff, da kommen zumindest nicht meine Messwerte raus...
Also der Strom fließt ja zuerst durch den 200er, dann durch den 100er und dann durch den 300er Widerstand, den Weg habe ich auch so bei meinen Rechenversuchen benutzt, kan aber nischt bei raus...
tuztiuoipoi
Gast





Beitrag tuztiuoipoi Verfasst am: 11. Sep 2010 23:07    Titel: Antworten mit Zitat

Die Lösung ist sehr (SEHR!!!!!!!!!) einfach. Schlissen wir den wurfel in Stromkreis. Dann weil die Aufbau symmetrisch ist die Spannungen in Punkten E, D, B sind gleich. Eben für Punkten H, F, C.
D.h., dass wie die Punkte EDB so wie Punkte HFC mit einem Draht verbinden können, ohne die Ströme zu ändern!
Aber jetzt können wir Widerstand leicht kalkulieren:
R=R1/3 + R2/6 + R3/3= 300/3+100/6+200/3=....
Entschuldige für mögliche Sprachfehler. Bin ein Russe.
Nice_price



Anmeldungsdatum: 05.09.2010
Beiträge: 4

Beitrag Nice_price Verfasst am: 12. Sep 2010 08:43    Titel: Antworten mit Zitat

Guten Morgen und schon einmal vielen Dank für die Antwort! Thumbs up!
Das zwischen den Punkt kein Spannungsunterschied ist hab ich dann, aber WARUM ist da keine? Und WIE kommt dann die Formel zusammen die du da aufgestellt hast?
Das Deutsch ist im Übrigen besser als bei so manchem Deutschen;)

Vielen Dank!
Torben
Junge



Anmeldungsdatum: 16.11.2009
Beiträge: 39

Beitrag Junge Verfasst am: 12. Sep 2010 10:35    Titel: Antworten mit Zitat

Erstmal, danke!
1. Wenn du rotierst mal den wurfel rund um die Axe AG auf +- 120 grad, kannst Du nicht mehr feststellen, wo Du früher war - im Punkt E, B oder D - die Aufbau ist symmetrisch. D.h, die Ströme, die fliessen über AE, AB und AD sind gliech und weil Rae =Rab=Rad, heisst es, dass die Spannungen Ue=Ub=Ud.
2. Jetzt können wir den Draht anlegen - kein Strom fliesst über den Draht, es scheint, als es keinen gäbe. Er ändert nichts (bezüglich R).
3. Doch jetz haben wir das Schema wie auf Abb. und jetz können wir den Widerschtand berechnen
Das war's



res.jpg
 Beschreibung:
 Dateigröße:  22.94 KB
 Angeschaut:  8530 mal

res.jpg


Nice_price



Anmeldungsdatum: 05.09.2010
Beiträge: 4

Beitrag Nice_price Verfasst am: 12. Sep 2010 20:18    Titel: Antworten mit Zitat

Ui, jetzt versteh sogar ich das=)

Liebe Grüße und vielen Dank für die Antworten!

Torben
isi1



Anmeldungsdatum: 03.09.2006
Beiträge: 2902
Wohnort: München

Beitrag isi1 Verfasst am: 13. Sep 2010 15:50    Titel: Re: Widerstandswürfel mit unterschiedlichen Widerständen Antworten mit Zitat

Nice_price hat Folgendes geschrieben:
Außerdem soll ich aber auch hier den Gesamtwiderstand berechnen, der mit meinen gemessenen ca.180 Ohm aber nicht so wirklich richtig zusammenpasst.Das Netzgerät zeigte bei diesem Würfel bei den geforderten 2V auch nur noch eine Stromstärke von 0,01A an.
Stimmt doch:

R = 300/3 + 200/3 + 100/6 = 183,33 Ω

2V / 183 Ω = 0,011A

_________________
Grüße aus München, isi •≡≈ ¹₁₂½√∠∞±∫αβγδεηκλπρσφω ΔΣΦΩ
Neue Frage »
Antworten »
    Foren-Übersicht -> Elektrik